Board logo

標題: 99 屏北高中 [打印本頁]

作者: weiye    時間: 2010-5-11 21:25     標題: 99 屏北高中

屏北高中也公佈題目了,

不過似乎是用直接掃描的,原始檔案有點大,

所以小弟又把它重新打過,

如有打錯字或數據打錯的話,希望網友能不吝告知,感謝。

另外,想看該校提供的原始題目的話,

可見 《點我》

附件: 99ppsh.pdf (2011-5-19 21:13, 192.63 KB) / 該附件被下載次數 13782
https://math.pro/db/attachment.php?aid=184&k=30610a234fe981f1e05bc71e4eea3ef5&t=1711716558

附件: 99ppsh(LaTeX_source).rar (2011-5-19 21:13, 2.65 KB) / 該附件被下載次數 12288
https://math.pro/db/attachment.php?aid=185&k=f8d67a1d7bdee9c9e451c783f89b353f&t=1711716558
作者: bugmens    時間: 2010-5-11 22:37

3.假設直角三角形的三個頂點分別為\( A=(0,0) \),\( B=(1,0) \)和\( C=(0,4) \),令\( Q=(x,y) \)為此三角形內部的一個點,試求點Q和點Q到三個頂點距離之和的最小值(即\( \vert\ Q-A \vert\ + \vert\ Q-B \vert\ + \vert\ Q-C \vert\ \)的最小值)
[提示]
費馬點,以AC為邊長作正三角形ACP,其中P點坐標為\( (-2 \sqrt{3},2) \),求\( \overline{PB} \)就是最小值



關於費馬點的題目我比較喜歡這題
\( x,y,z \)為正實數,\( \displaystyle \Bigg\{\ \matrix{9=x^2+y^2+xy \cr 16=y^2+z^2+yz \cr 25=z^2+x^2+zx } \),求\( x+y+z= \)?




3.如下圖,△ABC,\( ∠C=90^o \),\( \overline{AD}=\overline{DE}=\overline{EB} \),∠ACD=α,∠DCE=β,∠ECD=γ,求\( \displaystyle \frac{sin α \cdot sin γ}{sin β}= \)?
(徐氏數學2A P2.5-8)
(我的教甄準備之路 面積法,有相同圖形的類似題目)
https://math.pro/db/viewthread.php?tid=661&page=1#pid1112



8.△ABC中,\( ∠ABC=90^o \),\( \overline{AB}=1 \),若延長\( \overline{AC} \)到D,並使得\( \overline{AB}=\overline{CD}=1 \),若\( ∠CBD=30^o \),求\( \overline{AC} \)長。
(我的教甄準備之路 面積法,https://math.pro/db/viewthread.php?tid=661&page=1#pid1112)
作者: may    時間: 2010-5-12 21:36     標題: 回復 2# bugmens 的帖子

請教你提到費馬點的問題該怎麼解呢?謝謝。
作者: weiye    時間: 2010-5-13 13:47

引用:
原帖由 may 於 2010-5-12 09:36 PM 發表
請教你提到費馬點的問題該怎麼解呢?謝謝。
設 \( x,y,z \)為正實數,\( \displaystyle \left\{\ \matrix{9=x^2+y^2+xy \cr 16=y^2+z^2+yz \cr 25=z^2+x^2+zx }\right. \),求\( x+y+z= \)?

解答:

\( \displaystyle \left\{\ \matrix{3^2=x^2+y^2-2xy\cos120^\circ \cr 4^2=y^2+z^2-2yz\cos120^\circ \cr 5^2=z^2+x^2-2zx\cos120^\circ }\right. \)

令 \(\triangle ABC\) 滿足 \(AB=4, AC=3, BC=5\),

且 \(P\) 為 \(\triangle ABC\) 內部一點,滿足 \(PC=x, PA=y, PB=z, ∠ APB=∠ BPC=∠ CPA=120^\circ\),

題目要求 \(x+y+z=PC+PA+PB.\)

將 \(\triangle CAP\) 以 \(A\) 點為中心,往外旋轉 \(60^\circ\) 得 \(\triangle DAQ\),

則 \(CP=DQ, ∠ DQA=120^\circ\)。

在 \(\triangle APQ\) 中,因為 \(AP=AQ\) 且 \(∠ PAQ=60^\circ\),

得 \(\triangle PAQ\) 為正三角形,且 \(PQ=PA=y\) 且 \(∠ PQA=∠ QPA=60^\circ.\)

故,\(D,Q,P,B\) 四點共線,且 \(DB=DQ+QP+PB=PC+PA+PB=x+y+z.\)

利用畢氏定理,可得

\(DB=\sqrt{\left(\frac{3}{2}\right)^2+\left(\frac{3\sqrt{3}}{2}+4\right)^2}=\sqrt{25+12\sqrt{3}}.\)
作者: carrefourtaiwan    時間: 2010-5-20 20:13

請問第七題要如何解題呢?
還有第九題除了帶n=1,n=2的機率解聯立求a,b之外,不知是否還有其他辦法可解題?
(我是找出n=1和n=2時數字1出現偶數點的機率去解聯立求a,b)
謝謝老師們的幫忙!
作者: weiye    時間: 2010-5-20 20:38

第 7 題

若 \(m\in\mathbb{N}\),求 \(\displaystyle \lim_{n\to\infty} n^2\left(\frac{m}{n} - \frac{1}{n+1} - \frac{1}{n+2} - \cdots  - \frac{1}{n+m}\right)=?\)


解答:

\(\displaystyle \lim_{n\to\infty} n^2\left(\frac{m}{n} - \frac{1}{n+1} - \frac{1}{n+2} - \cdots  - \frac{1}{n+m}\right)\)

\(\displaystyle =\lim_{n\to\infty} n\left(m - \frac{n}{n+1} - \frac{n}{n+2} - \cdots  - \frac{n}{n+m}\right)\)

\(\displaystyle =\lim_{n\to\infty} n\left\{ \left(1- \frac{n}{n+1}\right) +\left(1-\frac{n}{n+2}\right)+\cdots +\left(1 - \frac{n}{n+m}\right)\right\}\)

\(\displaystyle =\lim_{n\to\infty} n\left( \frac{1}{n+1} +\frac{2}{n+2}+\cdots +\frac{m}{n+m}\right)\)

\(\displaystyle =\lim_{n\to\infty} \left( \frac{n}{n+1} +\frac{2n}{n+2}+\cdots +\frac{mn}{n+m}\right)\)

\(\displaystyle =\lim_{n\to\infty} \left( \frac{1}{1+\frac{1}{n}} +\frac{2}{1+\frac{2}{n}}+\cdots +\frac{m}{1+\frac{m}{n}}\right)\)

\(\displaystyle =\frac{1}{1+0} +\frac{2}{1+0}+\cdots +\frac{m}{1+0}\)

\(\displaystyle =1+2+\cdots+m=\frac{m\left(m+1\right)}{2}.\)






第 9 題:

若投擲 \(n\) 顆公正的骰子,有偶數顆為 \(1\) 點的機率是 \(\displaystyle \frac{1}{2}\left(a+b^n\right)\),求 \(\left(a,b\right)=?\)

解答:

設擲 \(n\) 顆公正的骰子,有偶數顆為 \(1\) 點的機率是 \(P(n)\),則有奇數顆是 \(1\) 的機率為 \(1-P(n).\)

先找出遞迴關係,

\(\displaystyle P(1)=\frac{5}{6}\) ,且當 \(\displaystyle n\geq2\) 時,\(\displaystyle P(n)=\frac{1}{6}\left(1-P(n-1)\right)+\frac{5}{6}P(n-1)\)

                \(\displaystyle \Rightarrow P(n)=\frac{1}{6}+\frac{2}{3}P(n-1)\)

先調整成形如 \(\displaystyle \left(P(n)+k\right)=\lambda\left(P(n-1)+k\right)\) 的形式,

\(\displaystyle P(n)-\frac{1}{2}=\frac{2}{3}\left(P(n-1)-\frac{1}{2}\right)\)

所以,

   \(\displaystyle P(n)-\frac{1}{2}=\frac{2}{3}\left(P(n-1)-\frac{1}{2}\right)\)

   \(\displaystyle P(n-1)-\frac{1}{2}=\frac{2}{3}\left(P(n-2)-\frac{1}{2}\right)\)

      \(\displaystyle \vdots\)

   \(\displaystyle P(2)-\frac{1}{2}=\frac{2}{3}\left(P(1)-\frac{1}{2}\right)\)

將上式乘起來,可得

\(\displaystyle P(n)-\frac{1}{2}=\left(\frac{2}{3}\right)^{n-1}\left(P(1)-\frac{1}{2}\right)\)

\(\displaystyle \Rightarrow P(n)=\frac{1}{2}+\left(\frac{2}{3}\right)^{n-1}\left(\frac{5}{6}-\frac{1}{2}\right)\)

    \(\displaystyle =\frac{1}{2}+\frac{1}{3}\left(\frac{2}{3}\right)^{n-1}\)

    \(\displaystyle =\frac{1}{2}+\frac{1}{2}\left(\frac{2}{3}\right)^{n}\)

    \(\displaystyle =\frac{1}{2}\left(1+\left(\frac{2}{3}\right)^{n}\right).\)

類似題目:

1. https://math.pro/db/thread-408-1-1.html

2. https://math.pro/db/thread-626-1-1.html
作者: carrefourtaiwan    時間: 2010-5-20 22:01

謝謝weiye老師
也很感謝您提供相關題目,獲益良多。
謝謝您
作者: mandy    時間: 2010-5-26 00:11     標題: 請問

請問填充第2 : .............有多少組可能的根使得\(g(x)\)可表成\(x^5+ax^4+bx^3+cx^2+dx+6=0\) , 如何求?
作者: weiye    時間: 2010-5-26 09:58

99 國立屏北高中教師甄試試題

第 2 題:

假設一個 \(5\) 次整係數多項式 \(g(x)\) 的根全為整數,試問共有多少組可能的根使得 \(g(x)\) 可以表成 \(g\left(x\right)=x^5+ax^4+bx^3+cx^2+dx+6\),其中 \(a,b,c,d\) 為整數.


解答:

由牛頓有理根定理(多項式的整係數一次因式檢驗法),

可知 \(g(x)=0\) 的整數根只有可能為 \(\pm1, \pm2, \pm3,\pm6\)

且因為五根之積為 \(-6\),

所以,

case i: 有兩根為 〝\(2,3\)〞 或 〝\(-2,-3\)〞時,

    只能搭配另外三根為 〝\(1,1,-1\)〞 或  〝\(-1,-1,-1\)〞 ,

   有 \(2\times2=4\) 組.


case ii: 有兩根為 〝\(-2,3\)〞 或 〝\(2,-3\)〞時,

    只能搭配另外三根為 〝\(1,1,1\)〞 或 〝\(1,-1,-1\)〞 ,

   有 \(2\times2=4\) 組.

case iii: 有一根為 \(-6\) 時,

    只能搭配另外四根為 〝\(1,1,1,1\)〞 或  〝\(-1,-1,1,1\)〞 或  〝\(-1,-1,-1,-1\)〞,

   有 \(1\times3=3\) 組.


case iv: 有一根為 \(6\) 時,

    只能搭配另外四根為 〝\(-1,1,1,1\)〞 或  〝\(-1,-1,-1,1\)〞,

   有 \(1\times2=2\) 組.

所以,共有 \(4+4+3+2=13\) 組.
作者: mandy    時間: 2010-5-27 21:06     標題: 請問

請問為什麼第9題的P(1)=5/6 ?  ..........一顆骰子有偶數顆為1點的機率不是0嗎?
作者: weiye    時間: 2010-5-27 21:30

引用:
原帖由 mandy 於 2010-5-27 09:06 PM 發表
請問為什麼第9題的P(1)=5/6 ?  ..........一顆骰子有偶數顆為1點的機率不是0嗎?
丟一顆骰子,可能出現的點數有 \(1,2,3,4,5,6\)

如果出現的點數是 \(1\),則有一顆(奇數顆)\(1\) 點.

如果出現的點數是 \(2,3,4,5,6\),則有零顆(偶數顆)\(1\) 點.

所以,丟一顆骰子時,出現有偶數顆 \(1\) 點的機率是 \(\displaystyle\frac{5}{6}.\)
作者: mandy    時間: 2011-3-22 18:16

請問計算證明題在單位圓上的極值如何求 ? 我用langrange multipler 做不出 .
作者: weiye    時間: 2011-3-22 19:15

計算證明題:試求函數 \(f(x,y)=x+y^2\) 在單位圓 \(\displaystyle\left\{(x,y)\Big|x^2+y^2=1\right\}\) 上的極值,並找出發生極值的點。

解一、

\(\displaystyle f(x,y)=x+y^2=x+\left(1-x^2\right)=-\left(x-\frac{1}{2}\right)^2+\frac{5}{4}\leq\frac{5}{4}\)

因為 \(y^2=1-x^2\geq0\Rightarrow -1\leq x\leq1\),

所以,

當 \(\displaystyle x=\frac{1}{2}\) 時,\(f(x,y)\) 有最大值為 \(\displaystyle\frac{5}{4}\),此時 \(\displaystyle(x,y)=(\frac{1}{2},\frac{\pm\sqrt{3}}{2})\)。

當 \(\displaystyle x=-1\) 時,\(f(x,y)\) 有最小值為 \(-1\),此時 \((x,y)=(-1,0)\)。




解二、

令 \((x,y)=(\cos\theta,\sin\theta)\),其中 \(0 \leq\theta<2\pi\),

則 .......(後半段跟"解一"差不多,表示成 \(\cos\theta\) 的一元二次方程式,再配方求極值。)



解三、

令 \(g(x,y,k)=x+y^2+k\left(x^2+y^2-1\right)\),

解聯立方程式 \(\displaystyle\frac{\partial g}{\partial x}=\frac{\partial g}{\partial y}=\frac{\partial g}{\partial k}=0\),

可得 \(\displaystyle(x,y,k)=\left(-1,0,\frac{1}{2}\right),\left(1,0,\frac{-1}{2}\right),\left(\frac{1}{2},\frac{\sqrt{3}}{2},-1\right),\left(\frac{1}{2},\frac{-\sqrt{3}}{2},-1\right)\)

可得極值 \(f(-1,0)=-1\),\(f(1,0)=1\),\(\displaystyle f(\frac{1}{2},\frac{\pm\sqrt{3}}{2})=\frac{5}{4}\)。








作者: mandy    時間: 2011-3-22 21:40

在請教第二題 : 一線段長15, 做成一正三角形......... ?
作者: weiye    時間: 2011-3-22 22:45

國立屏北高級中學 99 學年度第一次教師甄選(清華原住民教育實驗專班)
第 2 題:有一線段長 \(15\),以此線段做一正三角形、一正方形和一圓,請問三者面積和的最小值為何.

解答:

設正三角形邊長為 \(a\)、正方形邊長為 \(b\)、圓的半徑為 \(c\),

則 \(3a+4b+2\pi c=15\),

題目所求三者的面積為 \(\displaystyle \frac{\sqrt{3}a^2}{4}+b^2+\pi c^2\),

由科西不等式可得

  \(\displaystyle \left(\left(\frac{6}{\sqrt[4]{3}}\right)^2+\left(4\right)^2+\left(2\sqrt{\pi}\right)^2\right)\left(\left(\frac{\sqrt[4]{3} a}{2}\right)^2+\left(b\right)^2+\left(\sqrt{\pi} c\right)^2\right)\geq\left(3a+4b+2\pi c\right)^2\)

\(\displaystyle\Rightarrow \mbox{三者面積和}\geq\frac{225}{12\sqrt{3}+16+4\pi}\)

且當等號成立時,若且唯若 .....((這一段不寫了啦~~應該可以自己檢查等號是有可能成立的!感謝~:P))
作者: Ellipse    時間: 2011-3-24 00:02

引用:
原帖由 weiye 於 2010-5-13 01:47 PM 發表


設 \( x,y,z \)為正實數,\( \displaystyle \left\{\ \matrix{9=x^2+y^2+xy \cr 16=y^2+z^2+yz \cr 25=z^2+x^2+zx }\right. \),求\( x+y+z= \)?

解答: ...
這題若出x,y,z為實數,解題過程就很麻煩了
且x+y+z的值會有四組解...
作者: weiye    時間: 2011-3-24 08:49

引用:
原帖由 Ellipse 於 2011-3-24 12:02 AM 發表
這題若出x,y,z為實數,解題過程就很麻煩了
且x+y+z的值會有四組解...
還蠻有趣的,不限定要"正實數"的話,

用 WolframAlpha 解出來 \(x+y+z=\pm\sqrt{25+12\sqrt{3}}\) 或 \(\pm\sqrt{25-12\sqrt{3}}\)

(也就是 \(x+y+z\) 的四個可能值剛好會是 \(t^4-50t^2+193=0\) 的四個根。)

WolframAlpha:這裡
作者: mandy    時間: 2011-3-27 13:35

請問填充第4題如何求 ?
作者: weiye    時間: 2011-3-27 19:40

國立屏北高級中學 99 學年度第一次教師甄選(清華原住民教育實驗專班)
第 4 題:若地球方程式為 \(x^2+y^2+z^2=100\),且北緯 \(\theta\) 所在的平面方程式為 \(x+2y-2z=6\),請問南緯 \(3\theta\) 所在的平面為何?


解答:

地球半徑 \(=10\),

  球心到北緯 \(\theta\) 所在平面的距離\(\displaystyle=10\sin\theta=\frac{\left|0+2\cdot0-2\cdot0-6\right|}{\sqrt{1^2+2^2+\left(-2\right)^2}}\Rightarrow \sin\theta=\frac{1}{5}\)

  \(\displaystyle\Rightarrow\sin 3\theta=3\sin\theta-4\sin^3\theta=\frac{71}{125}\)

球心到南緯 \(3\theta\) 所在平面的距離 \(\displaystyle=10\sin3\theta=\frac{142}{25}\)

令所求平面方程式為 \(x+2y-2z=k\),則

                 \(\displaystyle\frac{142}{25}=\frac{\left|0+2\cdot0-2\cdot0-k\right|}{\sqrt{1^2+2^2+\left(-2\right)^2}}\)

                 \(\displaystyle k=\pm\frac{426}{25}\)

所得兩平面方程式 \(\displaystyle x+2y-2z=\pm\frac{426}{25}\) 為南緯及北緯\(3\theta\) 所在的平面,

其中距離 \(x+2y-2z=6\) 比較遠的平面是 \(\displaystyle x+2y-2z=-\frac{426}{25}\),

所以,南緯 \(3\theta\) 所在的平面方程式為\(\displaystyle x+2y-2z=-\frac{426}{25}\)。
作者: waitpub    時間: 2011-4-28 21:48

請問第六題:已知等腰三角形,若腰上的中線長為6。求三角形面積的最大值為何?
作者: 老王    時間: 2011-4-28 22:02

引用:
原帖由 waitpub 於 2011-4-28 09:48 PM 發表
請問第六題:已知等腰三角形,若腰上的中線長為6。求三角形面積的最大值為何?
假設AB=AC
重心為G
那麼GB=GC=4
三角形GBC面積最大在角BGC=90度的時候,此時面積為(GBC)=8
(ABC)=3*(GBC)=24
作者: bugmens    時間: 2011-4-28 22:07

同時間有三個人回文,解題還慢了一步,我還是補充資料就好了

6.已知等腰三角形,若腰上的中線長為6。求三角形面積的最大值為何?

若一等腰三角形的底邊上的高等於18cm,腰上的中線等於15cm。則這個等腰三角形的面積等於?
(初中數學競賽指導)

已知直角三角形的周長為\( 2+\sqrt{6} \),斜邊上的中線長為1,求這個三角形的面積?
雖然解題沒用到重心,但條件也有中線,故放在一起

101.1.10補充
有一個直角三角形,斜邊上的中線長為1,周長為\( 2+\sqrt{6} \),求此三角形的面積為?
(100卓蘭實驗高中,https://math.pro/db/thread-1165-1-1.html)

[ 本帖最後由 bugmens 於 2012-1-10 06:08 PM 編輯 ]

圖片附件: 初中數學競賽指導.gif (2011-4-28 22:18, 48.81 KB) / 該附件被下載次數 4878
https://math.pro/db/attachment.php?aid=323&k=4978055d4094d62e6f4b6329916917b6&t=1711716558



圖片附件: 全國中學數學競賽題解.gif (2011-4-28 22:18, 35.3 KB) / 該附件被下載次數 4656
https://math.pro/db/attachment.php?aid=324&k=23fb38124987f92ddc186906d6e938fc&t=1711716558


作者: weiye    時間: 2011-4-28 22:07

國立屏北高級中學 99 學年度第一次教師甄選(清華原住民教育實驗專班)

第 6 題:已知等腰三角形,若腰上的中線長為 \(6\),求三角形面積的最大值為何?



解答:


設底邊長為 \(2a\),與底邊垂直之高長為 \(b\),


則腰長為 \(\sqrt{a^2+b^2}\),


由三角形的中線定理,可得 \(\displaystyle \left(\sqrt{a^2+b^2}\right)^2+(2a)^2=2\left(\left(\frac{\sqrt{a^2+b^2}}{2}\right)^2+6^2\right)\),


整理可得 \(9a^2+b^2=144\)(或是,如下圖,套用畢氏定理亦可得此式,感謝 bugmens 提供此想法。)




由算幾不等式,可得 \(\displaystyle \frac{9a^2+b^2}{2}\geq\sqrt{9a^2\cdot b^2}\Leftrightarrow 72\geq 3ab\Leftrightarrow 24\geq \frac{2a\cdot b}{2}\)


所以,此三角的最大面積為 \(24\)。(此時,\(a=2\sqrt{2}\),\(b=6\sqrt{2}\)。)
作者: waitpub    時間: 2011-4-28 22:22

另外請教一下第三題,有想過要中線定理,也想利用ACE面積是ACD面積兩倍,然後用1/2absin去算,但就是算不出來!!
作者: weiye    時間: 2011-4-28 22:34     標題: 回復 24# waitpub 的帖子

國立屏北高級中學 99 學年度第一次教師甄選(清華原住民教育實驗專班)

第 3 題:如下圖, \(\triangle ABC,\, \angle C=90^\circ,\, \overline{AD}=\overline{DE}=\overline{EB},\, \angle ACD=\alpha,\, \angle DCE=\beta,\, \angle ECD=\gamma\),

       求 \(\displaystyle\frac{\sin\alpha\cdot\sin\gamma}{\sin\beta}=?\)




解答:

\(\displaystyle\frac{\sin\alpha\cdot\sin\gamma}{\sin\beta}\)


\(\displaystyle=\frac{1}{\displaystyle\frac{1}{2}\overline{AC}\cdot \overline{BC}}\cdot\frac{\displaystyle\frac{1}{2}\overline{CD}\cdot \overline{AC}\sin\alpha\cdot\frac{1}{2}\overline{CE}\cdot \overline{BC}\sin\gamma}{\displaystyle\frac{1}{2}\overline{CD}\cdot \overline{CE}\sin\beta}\)


\(\displaystyle=\frac{1}{\triangle ABC\mbox{面積}}\cdot\frac{\triangle ACD\mbox{面積}\cdot \triangle BCE\mbox{面積}}{\triangle CDE\mbox{面積}}\)


\(\displaystyle=\frac{1}{\triangle ABC\mbox{面積}}\cdot\frac{\displaystyle\frac{1}{3}\triangle ABC\mbox{面積}\cdot \frac{1}{3}\triangle ABC\mbox{面積}}{\displaystyle\frac{1}{3}\triangle ABC\mbox{面積}}\)


\(\displaystyle=\frac{1}{3}\)
作者: waitpub    時間: 2011-4-28 22:51

謝謝老師們的解說!
作者: ejo3vu84    時間: 2011-5-1 14:18

請教各位老師填充第三題最小值已求出
Q點我自己算是(   (6-√3)/11, (2√3-1)/11    )
跟答案差很多~~不曉得是不是我算錯還是觀念有誤
謝謝!!
作者: icesnow1129    時間: 2011-5-19 12:22

引用:
原帖由 weiye 於 2010-5-13 01:47 PM 發表


設 \( x,y,z \)為正實數,\( \displaystyle \left\{\ \matrix{9=x^2+y^2+xy \cr 16=y^2+z^2+yz \cr 25=z^2+x^2+zx }\right. \),求\( x+y+z= \)?

解答: ...
weiye大您好!!

謝謝您提供這麼好的方法!!
還想請教這邊P點(也就是費馬點)的實際座標,該如何下手?完全沒頭緒...

感謝回答~

想到了可以讓C點旋轉60度得D點座標
又因為B-P-D在同一條線上,所以可以做出P點的參數式
再利用向量PA、向量PB所夾120度可得P

但是這樣一來數字變的很複雜...有沒有其他的好方法呢?

[ 本帖最後由 icesnow1129 於 2011-5-19 04:05 PM 編輯 ]
作者: weiye    時間: 2011-5-19 17:13     標題: 回復 28# icesnow1129 的帖子

分別以 \(A, B\) 為中心,將 \(C\) 分別以逆時針、順時針旋轉 \(60^\circ\),

設旋轉後的兩點分別為 \(D\) 與 \(F\),

則 \(\overline{AF}\) 與 \(\overline{BD}\) 的交點即為 \(P\) 點。
作者: waitpub    時間: 2011-5-19 21:05

第一題的解答打錯了,應該是3的立方根<x<27。
作者: weiye    時間: 2011-5-19 21:08     標題: 回復 30# waitpub 的帖子

感謝,馬上修正打字錯誤~ ^__^
作者: loui315    時間: 2011-5-23 22:59

請問填充第十題如何解?
作者: weiye    時間: 2011-5-24 11:23     標題: 回復 32# loui315 的帖子

填充第 10 題

把圓心畫出來,然後把圓補起來,

看起來就如下圖,



用畢氏定理可求得 \(\overline{O_1O_2}\),

進而求得直角三角形 \(\triangle AO_1O_2\) 的高 \(\overline{AB}\),

可得 \(\overline{AP}\) 之值,

再求的 \(\angle AO_1P, \angle AO_2P\) 之值,

再來應該問題不大了。^__^
作者: WAYNE10000    時間: 2012-3-24 20:34     標題: 請教第1題

我令\(logx=k\)
但解不出\(0<x<1\)的這個範圍

盼請賜教
感激不盡
作者: weiye    時間: 2012-3-24 20:54     標題: 回復 34# WAYNE10000 的帖子

國立屏北高級中學 99 學年度第一次教師甄選(清華原住民教育實驗專班)

第 1 題:求不等式 \(\displaystyle \log_3 x + \log_x 3<\frac{10}{3}\) 的解。

解答:

令 \(k=\log_3 x,\)

則 \(\displaystyle k+\frac{1}{k}<\frac{10}{3}\)

\(\displaystyle \Rightarrow k+\frac{1}{k}-\frac{10}{3}<0\)

\(\displaystyle \Rightarrow \frac{3k^2+3-10k}{3k}<0\)

\(\Rightarrow (3k^2+3-10k)(3k)<0\)

\(\Rightarrow (3k-1)(k-3)(3k)<0\)

\(\displaystyle \Rightarrow k<0\) 或 \(\displaystyle \frac{1}{3}<k<3\)

\(\Rightarrow \log_3 x<0\) 或 \(\displaystyle \frac{1}{3}<\log_3 x<3\)

\(\Rightarrow 0<x<1\) 或 \(\displaystyle \sqrt[3]{3}<x<27\)
作者: mathca    時間: 2015-12-31 18:59     標題: 回復 19# weiye 的帖子

請問第4題,「北緯   所在的平面方程式」,這句話是甚麼意思(被文字限制住了),感謝。
作者: thepiano    時間: 2015-12-31 20:54     標題: 回復 36# mathca 的帖子

北緯 θ 度的橫切面方程式
作者: mathca    時間: 2015-12-31 22:35     標題: 回復 37# thepiano 的帖子

稍微理解之後,莫非他不是放在傳統三維座標上---> e1=(1,0,0)  e2=(0,1,0)  e3=(0,0,1) 為基底
而是放在其他 類似斜坐標上(應該是把座標軸旋轉一定角度吧?)
不然傳統座標上,比如說北緯60度,形成的就是 z=常數 的平面(截面是圓),
應該是這樣吧?
作者: thepiano    時間: 2016-1-1 21:07     標題: 回復 38# mathca 的帖子

本討論串 19# 有站長大的解答
作者: mathca    時間: 2016-1-1 21:28     標題: 回復 39# thepiano 的帖子

摁,感謝。之前因看不懂題目描述,所以提問。




歡迎光臨 Math Pro 數學補給站 (https://math.pro/db/) 論壇程式使用 Discuz! 6.1.0